Quantcast
  • Register
PhysicsOverflow is a next-generation academic platform for physicists and astronomers, including a community peer review system and a postgraduate-level discussion forum analogous to MathOverflow.

Welcome to PhysicsOverflow! PhysicsOverflow is an open platform for community peer review and graduate-level Physics discussion.

Please help promote PhysicsOverflow ads elsewhere if you like it.

News

PO is now at the Physics Department of Bielefeld University!

New printer friendly PO pages!

Migration to Bielefeld University was successful!

Please vote for this year's PhysicsOverflow ads!

Please do help out in categorising submissions. Submit a paper to PhysicsOverflow!

... see more

Tools for paper authors

Submit paper
Claim Paper Authorship

Tools for SE users

Search User
Reclaim SE Account
Request Account Merger
Nativise imported posts
Claim post (deleted users)
Import SE post

Users whose questions have been imported from Physics Stack Exchange, Theoretical Physics Stack Exchange, or any other Stack Exchange site are kindly requested to reclaim their account and not to register as a new user.

Public \(\beta\) tools

Report a bug with a feature
Request a new functionality
404 page design
Send feedback

Attributions

(propose a free ad)

Site Statistics

205 submissions , 163 unreviewed
5,054 questions , 2,207 unanswered
5,345 answers , 22,721 comments
1,470 users with positive rep
818 active unimported users
More ...

  Anomalous triangle vertex: divergencies and symmetry argument

+ 2 like - 0 dislike
373 views

Consider the triangle correlator of one axial-vector current $J_{\lambda 5}$ and two vector currents $J_{\mu}, J_{\nu}$ in a theory with a fermion with mass $m$:
$$
\Gamma_{\lambda \mu\nu}(q,k,p) = F\bigg[\langle 0|T(J_{\lambda 5}(x)J_{\mu}(y)J_{\nu}(z) )|0\rangle \bigg]_{q,k,p},
$$
where $F[]$ means the Fourier transformation. The momentum $q$ is $q = p+k$. By using the perturbation theory, it can be related to triangle diagram with massive fermion running in the loop.

Adler proposes to use tensor decomposition
$$
\Gamma_{\lambda\mu\nu}(q,k,p) = A(p,k,m)\epsilon_{\lambda \mu\nu\rho}k^{\rho}+ \epsilon_{\nu \lambda \sigma\rho}p^{\sigma}k^{\rho}(B(p,k,m)p_{\mu}+C(p,k,m)k_{\mu})\\ + \begin{pmatrix} k \leftrightarrow p \\ \mu \leftrightarrow \nu\end{pmatrix},
$$
following from lorentz covariance, parity symmetry and bose symmetry. This expansion has historical name Rosenberg expansion.

The ''form-factors'' $B, C$ have the dimension -2 in energy units, while $A$ has the dimension 0; this follows from the dimensionality of $\Gamma_{\lambda \mu\nu}$, which is 1. Therefore, since the form-factors are obtained by the loop integration, $A$ is potentially logarithmically divergent. However, actually it can be shown that it is finite assuming the vector-like current conservation. Really, one has $p^{\mu}\Gamma_{\lambda \mu \nu} = 0$, or
$$
\tag 1 A(p,k,m) = B(p,k,m)p^{2} +C(p,k,m)(p\cdot k)
$$
Therefore one knows that $\Gamma_{\lambda \mu \nu}$ is free from any divergences without any calculations, since now potentially diverged $A$ is expressed in terms of finite $B,C$ with finite pre-factors.

Let's now assume particular case $p = -k$ which corresponds to $q = 0$. For the vertex $\Gamma_{\lambda \mu \nu}$ one has the general form
$$
\Gamma_{\lambda \mu \nu}(0, k, -k) = D(k^{2},m^{2})\epsilon_{\lambda \mu\nu\sigma}k^{\sigma},
$$
where $D$ is dimension 0 and a priori contains logarithmic singularity.

My questions are the following:

  • is it possible to use the identity $(1)$ for the case $p = -k$ by considering first the general case of unrelated $k,p$ and then by taking the limit

$$
\Gamma_{\lambda \mu\nu}(0,k,-k) \equiv \lim_{p \to -k}\Gamma_{\lambda \mu\nu}(q,k,p)?
$$

  • if the answer on the previous question is no, is there any other way to ensure that $D$ contains or doesn't contain logarithmic divergence without direct computations of the loop integral?
asked Dec 9, 2017 in Theoretical Physics by NAME_XXX (1,060 points) [ revision history ]
edited Dec 9, 2017 by NAME_XXX

Your answer

Please use answers only to (at least partly) answer questions. To comment, discuss, or ask for clarification, leave a comment instead.
To mask links under text, please type your text, highlight it, and click the "link" button. You can then enter your link URL.
Please consult the FAQ for as to how to format your post.
This is the answer box; if you want to write a comment instead, please use the 'add comment' button.
Live preview (may slow down editor)   Preview
Your name to display (optional):
Privacy: Your email address will only be used for sending these notifications.
Anti-spam verification:
If you are a human please identify the position of the character covered by the symbol $\varnothing$ in the following word:
p$\hbar$ysicsOverflo$\varnothing$
Then drag the red bullet below over the corresponding character of our banner. When you drop it there, the bullet changes to green (on slow internet connections after a few seconds).
Please complete the anti-spam verification




user contributions licensed under cc by-sa 3.0 with attribution required

Your rights
...